Đến nội dung

Hình ảnh

Topic BẤT ĐẲNG THỨC ôn thi vào lớp 10 THPT 2017 - 2018

bất đẳng thức am-gm cauchy bunyakovski minskovski schwarz holder thcs

  • Please log in to reply
Chủ đề này có 299 trả lời

#1
Nguyenphuctang

Nguyenphuctang

    Sĩ quan

  • Banned
  • 499 Bài viết

Sau khi topic hình học, hệ phương trình, phương trình,... đã mở thì mình mở luôn topic này cho xôm.

 

Các bạn cứ vào đây tham gia thảo luận nhé.

 

:excl: Một số quy định khi tham gia TOPIC: 

  • Không đăng các bài còn hạn trên các tạp chí THTT, TTT2, Pi,.....
  • Vui lòng đánh số bài khi tham gia Topic, ghi nguồn rõ ràng (nếu không nhớ rõ thì để là sưu tầm).
  • Không văng tục, spam, dùng lời lẽ xúc phạm các thành viên.
  • Hạn chế hỏi bài tập về nhà.
  • Lời giải ưu tiên gọn nhẹ, sáng tạo phù hợp với THCS (Không giới hạn về kiến thức, phương pháp giải).

Mong mọi người ủng hộ Topic.

 

Sau đây là những bài toán đề xuất đầu tiên: 

 

Bài 1: (Thi thử vào 10 chuyên toán KHTN 2016-2017 đợt 1 vòng 2)

Cho a, b, c > 0; ab + bc + ca = 1. Chứng minh rằng:

$\sum \frac{1-a^{2}}{1+a^{2}}\leq \sum \frac{a}{\sqrt{1+a^{2}}}$

Bài 2: (JBMO 2016)

Cho a, b, c là các số thực dương. Chứng minh rằng:

$\sum \frac{8}{(a+b)^{2}+4abc}+a^{2}+b^{2}+c^{2}\geq \sum \frac{8}{a+3}$

 

Bài 3: (Nguyễn Phúc Tăng - Trần Quốc Anh - Tạp chí Toán học Rumania)

Cho a, b, c > 0 thỏa mãn $a^{2}+b^{2}+c^{2}=3$. Chứng minh rằng:

$\frac{a}{b}+\frac{b}{c}+\frac{c}{a}+a+b+c \geq 6$

 



#2
sharker

sharker

    Sĩ quan

  • Thành viên
  • 301 Bài viết

Bài 2: (JBMO 2016)

Cho a, b, c là các số thực dương. Chứng minh rằng:

$\sum \frac{8}{(a+b)^{2}+4abc}+a^{2}+b^{2}+c^{2}\geq \sum \frac{8}{a+3}$

 

 

 

${(a + b)^2} + 4abc \le 2({a^2} + {b^2})(c + 1)$
 $\to \frac{8}{{{{(a + b)}^2} + 4abc}} \ge \frac{4}{{({a^2} + {b^2})(c + 1)}}$
Ta lại có:
$\frac{4}{{({a^2} + {b^2})(c + 1)}} + \frac{{{a^2} + {b^2}}}{2} \ge \frac{4}{{\sqrt {2(c + 1)} }} \ge \frac{8}{{c + 3}}$
$BDT \Leftrightarrow c + 3 \ge 2\sqrt {2(c + 1)}  \Leftrightarrow {(c - 1)^2} \ge 0$
 $\to {a^2} + {b^2} + {c^2} + \sum {\frac{4}{{{{(a + b)}^2} + 4abc}} \ge \sum {\frac{8}{{a + 3}}} }$

Anh sẽ vẫn bên em dù bất cứ nơi đâu

Anh sẽ là hạt bụi bay theo gió

Anh sẽ là ngôi sao trên bầu trời phương Bắc

Anh không bao giờ dừng lại ở một nơi nào

Anh sẽ là ngọn gió thổi qua các ngọn cây

Em sẽ mãi mãi đợi anh chứ ??

will you wait for me forever


#3
Nguyenphuctang

Nguyenphuctang

    Sĩ quan

  • Banned
  • 499 Bài viết

Dũng tay nghề vẫn cao như ngày nào nhỉ ? :))

Đáp án của ban tổ chức:

16684293_409404069407904_420602229009975



#4
Mr Cooper

Mr Cooper

    Sĩ quan

  • Thành viên
  • 496 Bài viết

Bài 3: (Nguyễn Phúc Tăng - Trần Quốc Anh - Tạp chí Toán học Rumania)

Cho a, b, c > 0 thỏa mãn $a^{2}+b^{2}+c^{2}=3$. Chứng minh rằng:

$\frac{a}{b}+\frac{b}{c}+\frac{c}{a}+a+b+c \geq 6$

 

Ta đi chứng minh bất đẳng thức sau:

$\dfrac{a}{b}+\dfrac{b}{c}+\dfrac{c}{a} \geq \frac{9}{a+b+c}$

$\Leftrightarrow a+b+c + \sum \dfrac{a^2}{b} + \sum \dfrac{ab}{c} \ge 9$

Bổ đề: $\sum \dfrac{ab}{c} \ge \sqrt{3(a^2+b^2+c^2)}$ và $\sum \dfrac{a^2}{b} \ge \dfrac{(a+b+c)(a^2+b^2+c^2)}{ab+bc+ca}$

$\Rightarrow a+b+c + \sum \dfrac{a^2}{b} + \sum \dfrac{ab}{c} \ge a+b+c + \dfrac{(a+b+c)(a^2+b^2+c^2)}{ab+bc+ca} +\sqrt{3(a^2+b^2+c^2)}= a+b+c + \dfrac{3(a+b+c)}{ab+bc+ca} + 3$

Ta cần chứng minh:  $a+b+c + \dfrac{3(a+b+c)}{ab+bc+ca} \ge 6$

Đặt $a+b+c=x$ $\Rightarrow ab+bc+ca = \dfrac{x^2-3}{2}$

$x+ \dfrac{6x}{x^2-3} \ge 6 \Leftrightarrow (x-3)(x^2-3x+6) \ge 0$ 

Áp dụng BĐT vừa chứng minh được ta có:

$\dfrac{a}{b}+\dfrac{b}{c}+\dfrac{c}{a}+a+b+c \ge \dfrac{9}{a+b+c} + a+b+c \ge 6$


Bài viết đã được chỉnh sửa nội dung bởi Mr Cooper: 17-04-2017 - 21:02


#5
Iceghost

Iceghost

    Hạ sĩ

  • Thành viên
  • 79 Bài viết

 

Bài 3: (Nguyễn Phúc Tăng - Trần Quốc Anh - Tạp chí Toán học Rumania)

Cho a, b, c > 0 thỏa mãn $a^{2}+b^{2}+c^{2}=3$. Chứng minh rằng:

$\frac{a}{b}+\frac{b}{c}+\frac{c}{a}+a+b+c \geq 6$

 

Áp dụng bđt C-B-S và bđt AM-GM ta có

$$\dfrac{a}b + \dfrac{b}c + \dfrac{c}a + a + b + c = \dfrac{a^2}{ab} + \dfrac{b^2}{bc} + \dfrac{c^2}{ca} + a + b + c \geqslant \dfrac{(a+b+c)^2}{ab+bc+ca} + (a+b+c) \geqslant 2\sqrt{\dfrac{(a+b+c)^3}{ab+bc+ca}}$$

Ta sẽ chứng minh $(a+b+c)^3 \geqslant 9(ab+bc+ca)$. Theo bđt AM-GM

$$27(a^2+b^2+c^2)(ab+bc+ca)(ab+bc+ca) \leqslant (a^2+b^2+c^2+ab+bc+ca+ab+bc+ca)^3 = (a+b+c)^6$$

Hay $81(ab+bc+ca)^2 \leqslant (a+b+c)^6$. Ta có đpcm

Dấu '=' tại $a=b=c=1$



#6
Nguyenphuctang

Nguyenphuctang

    Sĩ quan

  • Banned
  • 499 Bài viết

Nếu thực sự chứng minh bổ đề trên ''cực khổ'' như vậy thì ta đi C-S rồi ép về 1 ẩn cho lẹ Hưng.

Lời giải bài 3: 

Áp dụng bất đẳng $Cauchy-Schwarz$: 

$VT= \sum \frac{a^{2}}{ab}+a+b+c \geq \frac{(a+b+c)^{2}}{ab+bc+ca}+a+b+c$

Đặt $p=a+b+c$ từ đó suy ra: $\sum ab = \frac{p^{2}-3}{2}$

$\Rightarrow VT\geq \frac{2p^{2}}{p^{2}-3}+p\geq 6\Leftrightarrow (p+2)(p-3)^{2}\geq 0$ (Luôn đúng)

Đẳng thức xảy ra khi và chỉ khi $a=b=c=1$.
Các khác chứng minh bổ đề gọn nhẹ:

Sau khi áp dụng C-S ta quy bổ đề về chứng minh: $(a+b+c)^{3} \geq 9(ab+bc+ca)$ $\Leftrightarrow \frac{(a+b+c)^{6}}{27}\geq(a^{2}+b^{2}+C^{2})(ab+bc+ca)^{2}$

Bất đẳng thức cuối hiển nhiên đúng vì theo $AM-GM$ ta có: 

$(\sum a^{2})(\sum ab)(\sum ab) \leq \left ( \frac{\sum a^{2} +2\sum ab}{3} \right )^{3}=\frac{(a+b+c)^{6}}{27}$

Như vậy có phải gọn nhẹ nhiều hơn không?  :icon6:



#7
Nguyenphuctang

Nguyenphuctang

    Sĩ quan

  • Banned
  • 499 Bài viết

Bài 2 và 3 đã xong và giờ là bài 1 sau đây là 4 cách khác nhau cho bài 1 của mình:

Cách 1, 2, 3: 

File gửi kèm  tang (1).pdf   79.54K   507 Số lần tải

Cách 4: (Hơi ảo diệu :D và mình mất khá nhiều thời gian để suy nghĩ ra cách này).

http://toanhocsocapc...nh-bat-ang.html

 



#8
Nguyenphuctang

Nguyenphuctang

    Sĩ quan

  • Banned
  • 499 Bài viết

Mình xin nhắc thêm là mình sẽ ít đăng những bài bất cũ và quá quen. Thay vào đó sẽ đăng những bài toán mới sáng tác,..... chủ yếu là rèn luyện tư duy tránh tư tưởng ''nhớ lại''. Vì hiện nay xu hướng là sáng tác đề không cho lại các bài toán cũ.

Bài toán đề xuất tiếp theo:

 

Bài 4: (Sưu tầm)

Cho a, b, c là các số thực không âm thỏa mãn không có 2 số nào đồng thời bằng không. Chứng minh rằng:

$\sum \frac{a^{2}}{b^{2}+c^{2}} \geq \frac{(a+b+c)^{2}}{2(ab+bc+ca)}$

 

Bài 5: (Olympic 30/4 2017)

Cho a, b, c > 0. Chứng minh rằng:

$\sum \frac{a^{4}+27}{b+c} \geq 2(a^{2}+b^{2}+c{^2})$

 

Bài 6: (Nguyễn Việt Hùng, HSGS)

Cho a, b, c > 0. Chứng minh rằng:

$\sum \frac{a^{2}}{b+c} \geq \frac{a^{3}+b^{3}+c^{3}}{6(ab+bc+ca)}+\frac{4}{9}(a+b+c)$


Bài viết đã được chỉnh sửa nội dung bởi Nguyenphuctang: 18-04-2017 - 15:09


#9
Nguyenphuctang

Nguyenphuctang

    Sĩ quan

  • Banned
  • 499 Bài viết

- mấy thánh làm em chóng hết cả mặt @@

Chào bạn! Bạn chóng mặt thì về nhà đi bệnh viện đi  :D  đừng vào đây ăn nói lung tung, spam. Bạn không giải bài thì mời bạn đi chỗ khác chơi vẫn còn nhiều người đang giải.  :icon6:

 

Cho đề vừa vừa thôi :v ôn thi lớp 10 mà

Có bài dễ bài khó bạn à thi vào lớp 10 cũng có nhiều bài khó mà bạn.
Mình hi vọng đây là lần đầu cũng là lần cuối topic xuất những những post như thế này! Mong các bạn đừng spam!



#10
TrBaoChis

TrBaoChis

    Hạ sĩ

  • Banned
  • 81 Bài viết
$\sum \frac{a^2}{b^2+c^2}$ $\geq$ $\sum\frac{a}{b+c}$  (Nâng cao pt 8 tập 2)
$\sum \frac{a}{b+c}$  $\geq$  $\frac{(a+b+c)^2)}{2(ab+bc+ca)}$  (C-S)


#11
viet9a14124869

viet9a14124869

    Trung úy

  • Thành viên
  • 903 Bài viết

Thể theo ý bạn Nguyenphuctang ,,, mình xin nêu lại cách làm bài 5 đã có tại đây https://diendantoanh...16-2017-lớp-10/

Đặt $a^2+b^2+c^2=x\Rightarrow \left\{\begin{matrix} \sum \frac{a^4}{b+c}\geq \frac{(a^2+b^2+c^2)^2}{2(a+b+c)}\geq \frac{x^2}{2\sqrt{3x}} & & \\ \sum \frac{27}{b+c}\geq \frac{243}{2(a+b+c)}\geq \frac{243}{2\sqrt{3x}} & & \end{matrix}\right.$

Do đó ta quay về chứng minh $\frac{x^2+243}{2\sqrt{3x}}\geq 2x \Leftrightarrow (x-27)^2(x^2+6x+81)\geq 0$ đúng !

Vậy ta có đpcm ,,,dẫu bằng xảy ra khi a=b=c=3 ^_^

 

p/s : Nếu không mọi người có thể giải theo cách dùng AM-GM thế này $a^4+27\geq 4a^3$ cũng được :biggrin:


Bài viết đã được chỉnh sửa nội dung bởi viet9a14124869: 18-04-2017 - 19:02

                                                                    SÓNG BẮT ĐẦU TỪ GIÓ

                                                                    GIÓ BẮT ĐẦU TỪ ĐÂU ?

                                                                    ANH CŨNG KHÔNG BIẾT NỮA 

                                                                    KHI NÀO...? TA YÊU NHAU .


#12
NHoang1608

NHoang1608

    Sĩ quan

  • Thành viên
  • 375 Bài viết

Bài 7 (@Phamphucduc): 
Cho $a,b,c$ là các số thực dương thỏa mãn $a^{2}+b^{2}+c^{2}\leq 3b$
Chứng minh rằng $\frac{1}{(a+1)^{2}}+\frac{4}{(b+2)^{2}}+\frac{8}{(c+3)^{2}}\geq 1$
 

Spoiler

 


Bài viết đã được chỉnh sửa nội dung bởi NHoang1608: 18-04-2017 - 14:25

The greatest danger for most of us is not that our aim is too high and we miss it, but that it is too low and we reach it.

----- Michelangelo----


#13
sharker

sharker

    Sĩ quan

  • Thành viên
  • 301 Bài viết

 

Bài 1: (Thi thử vào 10 chuyên toán KHTN 2016-2017 đợt 1 vòng 2)

Cho a, b, c > 0; ab + bc + ca = 1. Chứng minh rằng:

$\sum \frac{1-a^{2}}{1+a^{2}}\leq \sum \frac{a}{\sqrt{1+a^{2}}}$

 

 
$\sum {\frac{{1 - {a^2}}}{{1 + {a^2}}} \le \sum {\frac{a}{{\sqrt {1 + {a^2}} }}} } $
 $\Leftrightarrow \sum {\frac{{(1 - {a^2})}}{{(a + b)(a + c)}} = \sum {\frac{{(1 - {a^2})(b + c)}}{{(a + b)(b + c)(a + c)}}} }  \le \sum {\frac{a}{{\sqrt {(a + b)(a + c)} }} = }$ $\sum {\frac{{a\sqrt {b + c} }}{{\sqrt {(a + b)(b + c)(a + c)} }}}$
 $\Leftrightarrow \sum {a\sqrt {(b + c)(a + c)(a + b)(b + c)} }  \ge \sum {(1 - {a^2})(b + c} )$
 $\Leftrightarrow \sum {a\sqrt {({b^2} + 1)({c^2} + 1)} }  + \sum {{a^2}(b + c)}  \ge \sum {(b + c)}$ 
mà $\sum {a\sqrt {({b^2} + 1)({c^2} + 1)} }  + \sum {{a^2}(b + c)}  = \sum {a\left( {1 - bc + \sqrt {({b^2} + 1)(c + 1)} } \right)}  \ge \sum {a\left( {1 - bc + bc + 1} \right) = 2\sum a (dpcm)}$
p/s: á đù ,vừa ms ra thì lại giống cách 3 của bạn tăng :3 

Bài viết đã được chỉnh sửa nội dung bởi sharker: 17-04-2017 - 23:10

Anh sẽ vẫn bên em dù bất cứ nơi đâu

Anh sẽ là hạt bụi bay theo gió

Anh sẽ là ngôi sao trên bầu trời phương Bắc

Anh không bao giờ dừng lại ở một nơi nào

Anh sẽ là ngọn gió thổi qua các ngọn cây

Em sẽ mãi mãi đợi anh chứ ??

will you wait for me forever


#14
TrBaoChis

TrBaoChis

    Hạ sĩ

  • Banned
  • 81 Bài viết

Bài 7 (@Phamphucduc): 
Cho $a,b,c$ là các số thực thỏa mãn $a^{2}+b^{2}+c^{2}\leq 3b$
Chứng minh rằng $\frac{1}{(a+1)^{2}}+\frac{4}{(b+2)^{2}}+\frac{8}{(c+3)^{2}}\geq 1$
 

Spoiler

 

áp dụng BĐT  : $\frac{1}{a^2}$ + $\frac{1}{b^2}$ $\geq$ $\frac{8}{(a+b)^2}$ 


Bài viết đã được chỉnh sửa nội dung bởi TrBaoChis: 19-04-2017 - 22:15


#15
Mr Cooper

Mr Cooper

    Sĩ quan

  • Thành viên
  • 496 Bài viết

Bài 6: (Nguyễn Việt Hùng, HSGS)

Cho a, b, c > 0. Chứng minh rằng:

$\sum \frac{a^{2}}{b+c} \geq \frac{a^{3}+b^{3}+c^{3}}{6(ab+bc+ca)}+\frac{4}{9}(a+b+c)$

$(a+b)(b+c)(c+a) \ge \dfrac{8}{9}(a+b+c)(ab+bc+ca)$

$\Leftrightarrow (a+b)(b+c)(c+a) \ge 8abc$

Theo Bất Đẳng Thức Holder ta có:

$\sum \frac{a^{2}}{b+c}=\sum \frac{a^{3}}{ab+ca} \ge \dfrac{(a+b+c)^3}{6(ab+bc+ca)} = \dfrac{a^3+b^3+c^3}{6(ab+bc+ca)}+\dfrac{3(a+b)(b+c)(c+a)}{6(ab+bc+ca)} \ge  \dfrac{a^3+b^3+c^3}{6(ab+bc+ca)} + \dfrac{4}{9}(a+b+c)$


Bài viết đã được chỉnh sửa nội dung bởi Mr Cooper: 18-04-2017 - 11:48


#16
Mr Cooper

Mr Cooper

    Sĩ quan

  • Thành viên
  • 496 Bài viết

$\boxed{8}$ [Trần Quốc Anh] Cho $a,b,c$ là các số thực không âm thỏa mãn $a+b+c=2$. Chứng minh rằng:

\[(a^2+ab+b^2)(b^2+bc+c^2)(c^2+ca+a^2) \le 3 \]

$\boxed{9}$ [MOSP 2005] Cho $a,b,c \ge 0$ và không đồng thời bằng $0$ thỏa mãn $ab+bc+ca=1$. Chứng minh rằng:

\[ \dfrac{1}{a+b}+\dfrac{1}{b+c}+\dfrac{1}{c+a} \ge \dfrac{5}{2}\]



#17
Nguyenphuctang

Nguyenphuctang

    Sĩ quan

  • Banned
  • 499 Bài viết

Nghiên cú bài hình mình gởi lên trước tý nhé các anh e

 

Các anh em gõ word hay pdf tải về các bđt in đọc tham khảo tý nhé các Cao thủ

Những điều bạn đăng toàn SPAM không liên quan gì đến những vấn đề chúng tôi đang bàn luận. Tôi để ý và thay mặt các ban quản trị nhắc nhở bạn bớt SPAM, gõ LaTeX khi đăng bài .  :icon6:

Spoiler

Tôi xin trả lời góp ý của bạn: Mỗi bài tôi đề xuất đều suy nghĩ trước và chuẩn bị trước 1 lời giải ngắn gọn phù hợp với THCS nhẹ nhàng. Chẳng hạn như lời giải bài 1 tôi đăng bạn có đọc kĩ chưa? Dù sao cũng cám ơn bạn đã góp ý cho Topic. Mọi bàn luận liên quan đến Topic xin dừng lại ở đây.
Nếu có gì thắc mắc cứ nhắn tin riêng với tôi đừng đăng lên đây làm Topic ''mất đẹp''. Thân chào !


Bài viết đã được chỉnh sửa nội dung bởi Nguyenphuctang: 18-04-2017 - 13:06


#18
Nguyenphuctang

Nguyenphuctang

    Sĩ quan

  • Banned
  • 499 Bài viết

$\boxed{8}$ [Trần Quốc Anh] Cho $a,b,c$ là các số thực không âm thỏa mãn $a+b+c=2$. Chứng minh rằng:

\[(a^2+ab+b^2)(b^2+bc+c^2)(c^2+ca+a^2) \le 3 \]

Đề nghị Hưng lần sau đưa bài nhẹ nhàng hơn xíu . Bài 8 mình vẫn chưa nghĩ ra cách nào ngoài khai triển + Maple quy về Schur bậc 6. Mà cái này quá sức với THCS

Bài 4: (Sưu tầm)

Cho a, b, c là các số thực không âm thỏa mãn không có 2 số nào đồng thời bằng không. Chứng minh rằng:

$\sum \frac{a^{2}}{b^{2}+c^{2}} \geq \frac{(a+b+c)^{2}}{2(ab+bc+ca)}$

Bài 4: Ta có thể làm mạnh bất đẳng thức lên thành:

$\sum \frac{a^{2}}{b^{2}+c^{2}} \geq \frac{(\sum a^{2})^{2}}{2 \sum a^{2}b{2}} \geq \frac{(a+b+c)^{2}}{ab+bc+ca}$

Bài toán đề xuất tiếp theo:

 

Bài 10 : (T6 Tạp chí THTT số 474) 

Với $x$ là số thực thay đổi tìm giá trị nhỏ nhất của biểu thức:

f= \sqrt{x^{2}-2x+2} + \sqrt{x^{2}-8x+32}+ \sqrt{x^{2}-6x+25} + \sqrt{x^{2}-4x+20}+ \sqrt{x^{2}-10x+26}

 

Bài 11 : (Thi thử KHTN đợt 3 vòng 2)

Cho các số $a,b,c$ thỏa mãn $0\leq a,b,c \leq 2$ và $a+b+c=3$. Chứng minh rằng

$a^{2}+b^{2}+c^{2}\leq 5$

 

Bài 12 : (Korean MO ngày 2 2016)

Cho x, y, z là các số thực thỏa mãn $x^{2}+y^{2}+z^{2}=1$. Tìm Max:

$P=(x^{2}-yz)(y^{2}-xz)(z^{2}-xy)$

(Bài này cũng là đề thi HSG cấp thành phố Cao Lãnh 2017; Và cũng được anh Nguyễn Văn Huyện - Nguyenhuyen_AG đề xuất trên tạp chí THTT tháng 5 năm 2016)

 

Bài 13: (Lê Khánh Sỹ)

Cho a, b, c > 0. Chứng minh rằng:

$\sum \frac{a+b}{c} \geq 2.\sqrt{(a+b+c)(\frac{a}{bc} +\frac{b}{ca}+ \frac{c}{ab})}$

 

Các bạn hạn chế đưa link lời giải từ Topic khác. Nếu làm thì hãy gõ LaTeX cho mọi người dễ coi tránh rời rạc.


Bài viết đã được chỉnh sửa nội dung bởi Nguyenphuctang: 18-04-2017 - 16:15


#19
Mr Cooper

Mr Cooper

    Sĩ quan

  • Thành viên
  • 496 Bài viết

$\boxed{9}$ [MOSP 2005] Cho $a,b,c \ge 0$ và không đồng thời bằng $0$ thỏa mãn $ab+bc+ca=1$. Chứng minh rằng:

\[ \dfrac{1}{a+b}+\dfrac{1}{b+c}+\dfrac{1}{c+a} \ge \dfrac{5}{2}\]

$\boxed{9}$

Từ điều kiện $ab+bc+ca=1$ ta có được: $1 \ge ab , 1 \ge bc , 1 \ge ca$.Trong 3 số dương $a,b,c$ bất kỳ luôn có 2 số nằm cùng phía so với $1$, ta giả sử 2 số đó là $a,b$. Từ đó suy ra: \[(a-1)(b-1)\ge 0\]

\[\Rightarrow 1+ab \ge a+b\]

\[\Rightarrow 2 \ge 1+ab \ge a+b\]

Ta có: $ab+bc+ca=1 \Rightarrow c=\dfrac{1-ab}{a+b} \ge 0$

Thay $c=\dfrac{1-ab}{a+b}$ vào bất đẳng thức cần chứng minh ta được:

\[ \dfrac{1}{a+b} + \dfrac{a+b}{1+b^2} + \dfrac{a+b}{1+a^2} \ge \dfrac{5}{2} \]

\[ \Leftrightarrow \dfrac{1}{(a+b)^2} + \dfrac{1}{1+b^2} + \dfrac{1}{1+a^2} \ge \dfrac{5}{2(a+b)} \]

Theo $AM - GM$ ta có: \[\dfrac{1}{1+b^2} + \dfrac{1}{1+a^2} = 1 - \dfrac{b^2}{1+b^2}+1 - \dfrac{a^2}{1+a^2} \ge 1 - \dfrac{b}{2}+1 - \dfrac{a}{2}=2-\dfrac{a+b}{2}\] Ta cần chứng minh: \[\dfrac{1}{(a+b)^2}+2-\dfrac{a+b}{2} \ge \dfrac{5}{2(a+b)}\] \[\Leftrightarrow \dfrac{1}{(a+b)^3}+\dfrac{2}{a+b}-\dfrac{1}{2} \ge \dfrac{5}{2(a+b)^2} \]

Đặt $x=\dfrac{1}{a+b} \ge \dfrac{1}{2}$ , bất đẳng thức trên trở thành:

\[x^3 + 2x - \dfrac{1}{2} \ge \dfrac{5}{2}x^2 \Leftrightarrow \dfrac{1}{2}(x-1)^2(2x-1) \ge 0\] Bất đẳng thức trên đúng với mọi $x \ge \dfrac{1}{2}$.

Đẳng thức xảy ra khi: $a=b=1,c=0$ và các hoán vị tương ứng



#20
Mr Cooper

Mr Cooper

    Sĩ quan

  • Thành viên
  • 496 Bài viết

Bài 13: (Lê Khánh Sỹ)

Cho a, b, c > 0. Chứng minh rằng:

$\sum \frac{a+b}{c} \geq 2.\sqrt{(a+b+c)(\frac{a}{bc} +\frac{b}{ca}+ \frac{c}{ab})}$

 

Không mất tính tổng quát giả sử $b$ nằm giữa $a$ và $c$ .

Bất đẳng thức cần chứng minh tương đương với:

\[abc(a+b+c)+a^2b^2+b^2c^2+c^2a^2 \geq 2\sqrt{abc(ab+bc+ca)(ab^2+bc^2+ca^2)}\]

Áp dụng bất đẳng thức $AM-GM$ ta có: \[\begin{matrix} 2\sqrt{abc(ab+bc+ca)(ab^2+bc^2+ca^2)} & \leq ac(ab+bc+ca)+b(ab^2+bc^2+ca^2)\\ & =a^2bc+abc^2+a^2c^2+ab^3+b^2c^2+a^2bc \end{matrix}\]

Ta cần chứng minh: \[a^2b^2+ab^2c \geq ab^3+a^2bc \Leftrightarrow ab(c-b)(b-a) \geq 0\] 

Đẳng thức xảy ra khi và chỉ khi: $a = b = c$







Được gắn nhãn với một hoặc nhiều trong số những từ khóa sau: bất đẳng thức, am-gm, cauchy, bunyakovski, minskovski, schwarz, holder, thcs

1 người đang xem chủ đề

0 thành viên, 1 khách, 0 thành viên ẩn danh